QFT Lorentz transformation question

In summary: Lambda^T\eta\Lambda...is a statement of the exact equality of two matrices. In this context, the components on the left are written as \eta_{\mu\nu} and the components on the right are written as \Lambda^\mu{}_\sigma\eta^{\sigma\rho}\Lambda^\nu{}_\rho. Equating components, we get\eta_{\mu\nu}=\Lambda^\mu{}_\sigma\eta^{\sigma\rho}\Lambda^\nu{}_\rhoWe can take the derivative of both sides of this equation with respect to a rotation angle \theta, using the chain rule on both sides. On the right side, we use the fact that
  • #1
emob2p
56
1
Hi,
Today in QFT class, we examined an infinitesimal Lorentz transformation. From this we showed that a LT is represented by an anti-symmetric matrix. But this means that the diagonal elements should all be zeros. My question is how this reduces to the standard LT with gamma, gamma, one, one running along the diagonal and +/- v*gamma in the 12, 21 elements. Where is my thinking confused? Thanks.
 
Physics news on Phys.org
  • #2
emob2p said:
Hi,
Today in QFT class, we examined an infinitesimal Lorentz transformation. From this we showed that a LT is represented by an anti-symmetric matrix.

Wasn't it: the unit matrix + an infinitesimal antisymmetric matrix ?
 
  • #3
Yes that is correct. But then that would make the diagonal elements of their sum all 1. Where do the gammas come from?
 
  • #4
Beware

[tex] \Lambda^{\mu}{}_{\nu}=\delta^{\mu}_{\nu}+\epsilon^{\mu}{}_{\nu} [/tex]

BUT

[tex] \epsilon^{\mu\lambda}=\epsilon^{\mu}{}_{\nu}\eta^{\nu\lambda} [/tex]

AND

[tex] \epsilon^{\mu\lambda}=-\epsilon^{\lambda\mu} [/tex]

Do you see the difference ?

Daniel.
 
  • #5
I guess I see the difference, but how does that help the original question. Isn't eta(nu, lambda) just a the unity matrix with a -1 as the time-time element?
 
  • #6
The exponential map maps a Lie algrbra into a neighbourhood of the identity of a corresponding Lie group. An infinitesimal Lorentz transformation is an element of the Lorentz Lie algebra so(1,3), while a Lorentz transformation is an element of the Lorentz Lie group SO(1,3). A lie algebra element that has trace zero exponentiates to a Lie group element that has determinant +1 - in this case a proper Lorentz transformation.

Regards,
George
 
  • #7
As an example, consider the infinitesimal Lorentz tranformation that is zero everywhere except in 2x2 block in the upper left, which is

[tex] \left[ \begin{array}{ccc}
0 & w \\
w & 0 \\
\end{array} \right]
[/tex]

Exponentiate this 2x2 matrix (by diagonalizing first), and you'll see a standard boost expressed using hyperbolic trig.

Regards,
George
 
  • #8
emob2p said:
Yes that is correct. But then that would make the diagonal elements of their sum all 1. Where do the gammas come from?

Repeated multiplication by [itex](I+\delta)[/itex] leads to the Lorentz Transform when [itex]\delta[/itex] is symmetric.


[tex]\mbox{Lorentz Transform}\ \ =\ \ \lim_{\delta \rightarrow 0}
\left(\left|
\begin{array}{cccc}
1 & 0 & 0 & 0 \\
0 & 1 & 0 & 0 \\
0 & 0 & 1 & 0 \\
0 & 0 & 0 & 1
\end{array}
\right| +\left|
\begin{array}{cccc}
0 & \delta & 0 & 0 \\
\delta & 0 & 0 & 0 \\
0 & 0 & 0 & 0 \\
0 & 0 & 0 & 0
\end{array}
\right|\right)^{\psi/\delta}\ \ =\ \ \left(
\begin{array}{cccc}
\cosh{\psi} & \sinh{\psi} & 0 & 0 \\
\sinh{\psi} & \cosh{\psi} & 0 & 0 \\
0 & 0 & 1 & 0 \\
0 & 0 & 0 & 1
\end{array}
\right)
[/tex]


Repeated multiplication by [itex](I+\delta)[/itex] leads to Spatial Rotation when [itex]\delta[/itex] is anti-symmetric.


[tex]\mbox{Spatial Rotation}\ \ \ \ \ \ =\ \ \lim_{\delta \rightarrow 0}
\left(\left|
\begin{array}{cccc}
1 & 0 & 0 & 0 \\
0 & 1 & 0 & 0 \\
0 & 0 & 1 & 0 \\
0 & 0 & 0 & 1
\end{array}
\right| +\left|
\begin{array}{cccc}
0 & 0 & 0 & 0 \\
0 & 0 & \delta & 0 \\
0 & -\delta & 0 & 0 \\
0 & 0 & 0 & 0
\end{array}
\right|\right)^{\phi/\delta}\ \ =\ \ \left(
\begin{array}{cccc}
1 & 0 & 0 & 0 \\
0 & \cos{\phi} & -\sin{\phi} & 0 \\
0 & \sin{\phi} & \cos{\phi} & 0 \\
0 & 0 & 0 & 1
\end{array}
\right)
[/tex]



The angle [itex]\phi[/itex] and the boost [itex]\psi[/itex] are proportional to "the number of times you multiply"
The boost [itex]\psi[/itex] is related to the speed v by:

[tex]\tanh{(v/c)} = \psi[/tex]

Substitute this in the Lorentz Transform matrix and you get the usual form with the gammas.



Regards, Hans
 
Last edited:
  • #9
Won't those equalities only hold as psi and phi go to infinity? In other words, don't you get the eqaulities only after an infinite number of infinitesimal transformations?
 
  • #10
emob2p said:
Won't those equalities only hold as psi and phi go to infinity? In other words, don't you get the eqaulities only after an infinite number of infinitesimal transformations?

Correct, it's a limit, so you should read [tex]\psi/\delta[/tex] and [tex]\phi/\delta[/tex] in the exponents with [tex]\delta \rightarrow 0[/tex]

Regard, Hans
 
  • #11
can anybody kindly suggest some link or book for understanding lorentz group and lorentz from scratch.
 
  • #12
tayyaba aftab said:
can anybody kindly suggest some link or book for understanding lorentz group and lorentz from scratch.
Some other text I read referenced Ohnuki, but it's out of print, so you'd have to buy it used or look for it in a library. This one by Anadijiban Das looks good too. I haven't read either of these books, so I can only say that they appear to contain what you're looking for (but you should look inside and find out for yourself). I learned most of this stuff by reading chapter 2 of Weinberg's QFT book. You can definitely learn a lot from there too.
 
Last edited:
  • #13
If we view the components of [itex]\Lambda[/itex] as functions of six parameters, representing a velocity difference and a rotation, chosen so that we get the identity transformation when all parameters are 0, we can Taylor expand Lambda like this:

[tex]\Lambda(\theta)=\Lambda(0)+\theta^a\frac{\partial}{\partial\theta^a}\bigg|_0 \Lambda(\theta)+\mathcal O(\theta^2)[/tex]

where [itex]\mathcal O(\theta^2)[/itex] represents all terms of second order and higher. Now let's define

[tex]\omega=\theta^a\frac{\partial}{\partial\theta^a}\bigg|_0 \Lambda(\theta)[/tex]

and use this Taylor expansion in the equation that defines a Lorentz transformation:

[tex]\eta=\Lambda^T\eta\Lambda=(I+\omega+\mathcal O(\theta^2))^T\eta(I+\omega+\mathcal O(\theta^2))=\eta+\omega^T\eta+\eta\omega+\mathcal O(\theta^2)[/itex]

Since this holds for all [itex]\theta[/itex], the nth order terms on the right must be equal to the nth order terms on the left for all n. In particular, this means that

[tex](\eta\omega)^T=-\eta\omega[/tex]

Note that this is an exact equality, even though it came from a Taylor expansion.

Also, keep this in mind:
Fredrik said:
Recall that the components on row [itex]\mu[/itex], column [itex]\nu[/itex] of the matrices

[tex]\Lambda, \Lambda^T, \eta, \eta^{-1}, \omega[/tex]

are written as

[tex]\Lambda^\mu{}_\nu, \Lambda^\nu{}_\mu, \eta_{\mu\nu}, \eta^{\mu\nu}, \omega^\mu{}_\nu[/tex]

and that [itex]\eta^{-1}[/itex] and [itex]\eta[/itex] and used to raise and lower indices.
This means that [itex]\omega_{\mu\nu}[/itex] are actually the components of [itex]\eta\omega[/itex], not [itex]\omega[/itex]. That's why it's anti-symmetric.
 
Last edited:
  • #14
We can also Taylor expand the operator [itex]U(\Lambda))[/itex] that represents the Lorentz transformation [itex]\Lambda[/itex]:

[tex]U(\Lambda(\theta))=U(\Lambda(0))+\theta^a\frac{\partial}{\partial\theta^a}\bigg|_0 U(\Lambda(\theta))+\mathcal O(\theta ^2)[/tex]

[tex]=I+\theta^a\frac{\partial}{\partial\Lambda^\mu{}_\nu}\bigg|_I U(\Lambda) \frac{\partial}{\partial\theta^a}\bigg|_0\Lambda^\mu{}_\nu+\mathcal O(\theta ^2)[/tex]

[tex]=I+\omega^\mu{}_\nu\frac{\partial}{\partial\Lambda^\mu{}_\nu}\bigg|_I U(\Lambda)+\mathcal O(\theta ^2)[/tex]

Now define

[tex]X_\mu{}^\nu=\frac{\partial}{\partial\Lambda^\mu{}_\nu}\bigg|_I U(\Lambda)[/tex]

and note that

[tex]\omega^\mu{}_\nu X_\mu{}^\nu=\eta^{\mu\rho}\omega_{\rho\nu}\eta_{\mu\sigma} X^{\sigma\nu}=\omega_{\rho\nu}\delta^\rho_\sigma X^{\sigma\nu}=\omega_{\sigma\nu}X^{\sigma\nu}[/tex]

[tex]=\omega_{\sigma\nu}\left(\frac{X^{\sigma\nu}+X^{\nu\sigma}}{2}+\frac{X^{\sigma\nu}-X^{\nu\sigma}}{2}\right)=\omega_{\sigma\nu}\left(\frac{X^{\sigma\nu}-X^{\nu\sigma}}{2}\right)[/tex]

[tex]=\frac{i}{2}\omega_{\sigma\nu}(-i (X^{\sigma\nu}-X^{\nu\sigma}))[/tex]

(Recall that we always have [itex]A_{\mu\nu}S^{\mu\nu}=0[/itex] when A is anti-symmetric and S is symmetric).

This means that we can write

[tex]U(\Lambda(\theta))=I+\frac{i}{2}\omega_{\mu\nu}J^{\mu\nu}+\mathcal O(\theta^2)[/tex]

if we define

[tex]J^{\mu\nu}=-i (X^{\mu\nu}-X^{\nu\mu})[/tex]
 
  • #15
tayyaba aftab said:
can anybody kindly suggest some link or book for understanding lorentz group and lorentz from scratch.

At what level and with respect to what? Classical relativity? Quantum field theory?

A nice book is Relativity, Groups, and Particles: Special Relativity and Relativistic Symmetry in Field and Particle Physics by Roman U. Sexl and Helmuth K. Urbantke,

https://www.amazon.com/dp/3211834435/?tag=pfamazon01-20.
 
Last edited by a moderator:
  • #16
orignally posted by George Jones
"At what level and with respect to what? Classical relativity? Quantum field theory?

A nice book is Relativity, Groups, and Particles: Special Relativity and Relativistic Symmetry in Field and Particle Physics by Roman U. Sexl and Helmuth K. Urbantke,

https://www.amazon.com/Relativity-Gro...4394610&sr=1-1&tag=pfamazon01-20. "

i want to start from classical to qft.so that i make my concepts very clear.
 
Last edited by a moderator:

1. What is a Lorentz transformation in quantum field theory (QFT)?

A Lorentz transformation is a mathematical tool used in QFT to describe how quantities such as energy, momentum, and time change when viewed from different reference frames. It is based on the principles of special relativity and is used to understand the behavior of particles moving at high speeds.

2. How does the Lorentz transformation affect the equations in QFT?

The Lorentz transformation affects the equations in QFT by introducing a new set of mathematical rules for performing calculations in different reference frames. These rules take into account the relativistic effects of time dilation and length contraction, and allow for consistent predictions of physical phenomena at high speeds.

3. What is the significance of the Lorentz transformation in QFT?

The Lorentz transformation is crucial in QFT because it allows for the consistent formulation of relativistic quantum mechanics. It is a fundamental tool in understanding the behavior of particles at high speeds and has been confirmed by numerous experimental observations.

4. Can the Lorentz transformation be applied to all physical systems in QFT?

Yes, the Lorentz transformation can be applied to all physical systems in QFT, as long as they are described by relativistic quantum mechanics. This includes systems such as electrons, photons, and other particles that move at high speeds.

5. Are there any limitations to the Lorentz transformation in QFT?

While the Lorentz transformation is a powerful and widely applicable tool in QFT, it does have some limitations. It is only valid for systems that obey the laws of special relativity and cannot be used to describe gravitational effects. Additionally, it is not applicable to systems that are described by quantum mechanics but do not exhibit relativistic behavior.

Similar threads

Replies
6
Views
1K
  • Advanced Physics Homework Help
Replies
8
Views
739
  • Quantum Physics
Replies
1
Views
1K
  • Quantum Physics
Replies
3
Views
2K
  • Special and General Relativity
Replies
5
Views
956
  • Advanced Physics Homework Help
Replies
9
Views
1K
  • Quantum Physics
Replies
13
Views
1K
Replies
1
Views
1K
  • Special and General Relativity
Replies
6
Views
951
  • Special and General Relativity
Replies
30
Views
2K
Back
Top